COME ON!! What’s the answer 53 points!

COME ON!! Whats The Answer 53 Points!

Answers

Answer 1

Answer:

X : -6, 3, 15, -12

Y : 11, 5, -3, 15

Step-by-step explanation:

Hey there!

To find the y values we need to plug the x values into th given equation,

y = -2/3x + 7

-6 ⇒ 11

15 ⇒ -3

Now we have a x value missing to that that we work backwards,

we plug in 15 for y in the given equation,

15 ⇒-12

Hope this helps :)

Answer 2

Step-by-step explanation:

[tex]y = - \frac{2}{3} x + 7[/tex]

Substituting the values of x into the expression we have

For x = - 6

[tex]y = - \frac{2 }{3} ( - 6) + 7[/tex]

y = 11

For y = 5

Make x the subject

[tex]5 = - \frac{2}{3} x + 7[/tex]

Multiply through by 3

15 = - 2x + 21

-2x = 15 - 21

- 2x = - 6

Divide both sides by - 2

x = 3

For x = 15

[tex]y = - \frac{2}{3} (15) + 7[/tex]

y = - 10 + 7

y = - 3

For y = 15

[tex]15 = - \frac{2}{3} x + 7[/tex]

Multiply through by 3

45 = - 2x + 21

- 2x = 45 - 21

- 2x = 24

Divide both sides by - 2

x = - 12

Hope this helps you


Related Questions

If P(x)= x2 + x + 3 and Q(x) = 9x2 - 3, find P(8).
P(8)=D
(Type an integer or a fraction.)

Answers

Here you go! You actually said p(8) twice so I thought you meant Q(8). Please if the equations are wrong update me with the right equation
p(8) should equal 27

Which temperature during the week is the closest to 0? Explain how you decided this using absolute value.





Which temperature during the week is farthest from 0? Explain how you decided this using absolute value.





Which is the coldest temperature? Explain how you determined this using the number line.





Which is the warmest temperature? Explain how you determined this using the number line.





Part 2: Weather Report

Use the information from the analysis and develop a weather report. You can develop a flyer or make a presentation, video or audio recording of yourself, or script. Use your creativity! Make sure to include the information from all six questions.

Answers

we need a picture of the weather data

Help pls!! ASAP
Simplify.

(1+1/3)^2 −2/9



Enter your answer, as a simplified fraction, in the box .

Answers

Answer:

14/9

Step-by-step explanation:

(1+1/3)^2 −2/9

Add the terms in the parentheses

(3/3+1/3)^2 −2/9

(4/3)^2 −2/9

Exponents

16/9 - 2/9

14/9

For this problem, PEMDAS is really important, because we need to make sure that we are doing things in the correct order.

Parentheses

Exponents

Multiplication

Division

Addition

Subtraction

The first thing we need to do is the math inside the parentheses:

(1 + 1/3) = 1 + 1/3 = 4/3

Next, we square the parentheses:

(4/3)² = 16/9

Now, all we need to do is subtract!

16/9 - 2/9 = 14/9

If you need to make it a proper fraction, your answer is 1 5/9, but if you leave it improper then 14/9.

The graph below shows the relationship between the number of months different students practiced baseball and the number of games they won:

The title of the graph is Baseball Games. On x axis, the label is Number of Months of Practice. On y axis, the label is Number of Games Won. The scale on the y axis is from 0 to 22 at increments of 2, and the scale on the x axis is from 0 to 12 at increments of 2. The points plotted on the graph are the ordered pairs 0, 1 and 1, 3 and 2, 5 and 3, 9 and 4, 10 and 5, 12 and 6, 13 and 7, 14 and 8,17 and 9, 18 and 10,20. A straight line is drawn joining the ordered pairs 0, 1.8 and 2, 5.6 and 4, 9.2 and 6, 13 and 8, 16.5 and 10, 20.5.

Part A: What is the approximate y-intercept of the line of best fit and what does it represent? (5 points)

Part B: Write the equation for the line of best fit in slope-intercept form and use it to predict the number of games that could be won after 13 months of practice. Show your work and include the points used to calculate the slope. (5 points)

Answers

Answer:

(A) The y-intercept of the line is 1.82.

(B) The number of games that could be won after 13 months of practice is 26.

Step-by-step explanation:

The data from the provided graph is:

X Y

0 1

1 3

2 5

3 9

4 10

5 12

6 13

7 14

8 17

9 18

10 20

Here,

X : Number of Months of Practice

Y : Number of Games Won

(A)

Compute the y-intercept of the line as follows:

[tex]a=\frac{\sum Y\cdot \sum X^{2}-\sum X\cdot \sum XY}{n\cdot \sum X^{2}-(\sum X)^{2}}[/tex]

   [tex]=\frac{122\cdot 385-55\cdot 814}{11\cdot 385-(55)^{2}}\\\\=1.818\\\\\approx 1.82[/tex]

The y-intercept of the line is 1.82.

The y-intercept is the average value of the dependent variable, here the number of games won, when the value of the independent variable, here number of months of practice, is 0.

So, a y-intercept of 1.82 indicates that on average 1.82 can be won if the number of months of practice is 0.

(B)

Compute the slope as follows:

[tex]b=\frac{n\cdot \sum XY-\sum X\cdot \sum Y}{n\cdot \sum X^{2}-(\sum X)^{2}}[/tex]

  [tex]=\frac{11\cdot 814-55\cdot 122}{11\cdot 385-(55)^{2}}\\\\=1.855\\\\\approx 1.86[/tex]

The equation for the line of best fit in slope-intercept form  is:

[tex]y=1.82+1.85x[/tex]

Predict the number of games that could be won after 13 months of practice as follows:

[tex]y=1.82+1.85x[/tex]

  [tex]=1.82+(1.85\timex 13)\\=25.87\\\approx 26[/tex]

Thus, the number of games that could be won after 13 months of practice is 26.

Which numbers can be classified as rational? Select all that apply.

5/6

[tex] \sqrt{11} [/tex]
6.565656...

0.23

0.32416

-5 3/8

Answers

Answer:

5/6, 0.23, -5 3/8

Step-by-step explanation:

The answer is 5/6 with .

when the equation is graphed is it linear or nonlinear ??

Answers

Answer:

Nonlinear

Step-by-step explanation:

This is nonlinear because its parent function is y = [tex]x^{2}[/tex]. The parent function of a linear equation is y = x.

I hope this helps :))

The answer is nonlinear

What is g(h(10))? (images)

Answers

Answer:

2 sqrt(2)

Step-by-step explanation:

h(x) = 2x-8

g(x) = sqrt( x-4)

g(h(10))

First find h(10)

h(10) = 2*10 -8 = 20-8 =12

Then find g(h(10) = g(12) = sqrt( 12-4)

                                = sqrt(8) = sqrt(4*2) = sqrt(4) sqrt(2) = 2 sqrt(2)

Answer:

a

Step-by-step explanation:

edge 2021


Sasha deposited $1,400 in an account that pays 1.5% simple interest. She created a graph of her
account balance equation (A = Prt + P) with a slope of 21.

true or false?

Answers

Answer:

The answer is False.

I would agree. It would be false

Suppose that the values of x, y, and z are given by x=2/3 y=5/7 z= -11/3 What is xy/z

Answers

Answer:

The answer is -10/77 .

Step-by-step explanation:

In order to find the value of xy/z, you have to substitute the value of x, y and z into the expression :

[tex] \frac{xy}{z} = xy \div z[/tex]

[tex]( \frac{2}{3} \times \frac{5}{7}) \div - \frac{11}{3} [/tex]

[tex] = \frac{10}{21} \times - \frac{3}{11} [/tex]

[tex] = \frac{10}{7} \times - \frac{1}{11} [/tex]

[tex] = - \frac{10}{77} [/tex]

I think the answer is -10/77 I really hope this helps if not I’m sry

14+12= -15x+2x need help thanks

Answers

Answer:

  x = -2

Step-by-step explanation:

Collect terms:

  26 = -13x

Divide both sides of the equation by the coefficient of x:

  26/-13 = x = -2

The solution is x = -2.

Answer:

-2

Step-by-step explanation:

14+12=-15x+2x

26=-13x

x=26/-13

x=-2

All changes saved
2. The ratio of "D"s to "A"s in the school was 8 to 21. If there were 572 As in the school this term, how many Ds
were there?

Answers

Answer:

218

Step-by-step explanation:

572/21 = 27.23

8 x 27.23 = 217.9

Answer Check:

21/8 = 2.6

572/218 =2.6

8/21 = .38

218/572 = .38

The answer is 217.90 rounded is 218



First you divide 572/21
Next you get the quotient of 572/21 and multiple it by 8
Once you do that you should get your answer

Find the equation of a line that is perpendicular to line g that contains (P, Q).

coordinate plane with line g that passes through the points negative (3, 6) and (0, 5)

3x − y = 3P − Q
3x + y = Q − 3P
x − y = P − Q
x + y = Q − P

Answers

Answer:

x-y is parallel, im confused on what your asking for and what you mean by "negative"

Step-by-step explanation:

The  equation of a line that is perpendicular to line g that contains (P, Q). is 3x − y = 3P − Q

What is Equation of line?

The general form of the equation of a line with a slope m and passing through the point (x1, y1) is given as: y - y1 = m ( x- x1)

Further, this equation can be solved and simplified into the standard form of the equation of a line.

Given:

Line g passes through (3,6) and (0,5).

Slope of lone= y2 - y1/ (x2 - x1)

Perpendicular lines have opposite, reciprocal slopes, so negative change in x over change in y.

slope of line= -(-3 - 0)/(6 - 5)

                      = - -(-3)/1 =

slope of line = 3

Now, Two lines are perpendicular if they have the same slope.

Line parallel to line g has a slope of 1. Since it passes through (P, Q),

y - y1 = m ( x- x1)

y- Q =3 ( x- P)

y- Q = 3x- 3P

3x − y = 3P − Q

Learn more about equation of line here:.https://brainly.com/question/20519388

#SPJ2

For the equation, determine whether the equation is true for all, some, or no values of x.

Explain how you know.

Answers

Answer:

You can check if an equation is true for all some or no values of x by simply to  try and solve the equation if you get something like x = x, then the equation  is true for all, if you get a specific answer like x=3, then it is true to some values, in simple terms. Try and solve the equation there is no other way to check unless you solve, I don't know if there is another way but this is just my intution.  

Step-by-step explanation:

Answer:

111

Step-by-step explanation:

111

In the triangles, TR = GE and SR = FE.


Triangles S T R and F G E are shown. Angle S R T is 56 degrees. Angle F E G is 42 degrees. Sides T R and G E are congruent. Sides S R and F E are congruent.

if Line segment G F = 3.2 ft, which is a possible measure of Line segment T S?

Answers

Answer:

3.2 ft

Step-by-step explanation:

the only thing you need to know is that congruent means that all the side and the same and the shape can be flipped and still look the same so the measure of the line segment is 3.2ft

Answer:

3.2 ft

Step-by-step explanation:

The answer above is correct.

Name four fractions between 5/11 and 5/6

Answers

5/10, 5/9, 5/8, 5/7.

These are all between 5/11 and 5/6.

Answer:

6/11,  20/33,  2/3,  25/33  (answers vary)

Step-by-step explanation:

First, we need to convert both of these fractions to the same denominator. To do that, we need to find the least common denominator for both fractions.

6: 6, 12, 18, 24, 30, 36, 42, 48, 54, 60, 66, 72

11: 11, 22, 33, 44, 55, 66, 77

The LCM of 6 and 11 is 66. (6 * 11 = 66)

Now, we multiply 5/6 by 11/11 and 5/11 by 6/6.

[tex]\frac{5}{6} *\frac{11}{11}=\frac{55}{66}\\\\\\\frac{5}{11} *\frac{6}{6} =\frac{30}{66}[/tex]

Now, we can just choose four numbers between 30 and 55 and get our answer (remember to simplify!).

[tex]\frac{36}{66} =\frac{6}{11}[/tex]

[tex]\frac{40}{66}=\frac{20}{33}[/tex]

[tex]\frac{44}{66} =\frac{2}{3}[/tex]

[tex]\frac{50}{66} =\frac{25}{33}[/tex]

Step 2 Using Proportional Relationships

Answers

Answer:

a ( 567 miles to unit)

b( looks like 2 units so i would say 1134 miles

Answ: the scale is

Step-bstep explanatio

help meplz i want help i bad

Answers

Answer:

.00002

Step-by-step explanation:

2 * 10 ^-5

Move the decimal 5 places to the left since the exponent is negative

2.

We will need to add zeros on the left  Add 4 zeros since we can move it one place already

.00002

Answer:

(D) 0.00002

Step-by-step explanation:

Let's first forget about the 2 in the expression and focus on [tex]10^{-5}[/tex].

If we have 10 to a positive number, that many times the decimal place will move to the right. It's the opposite for 10 to the power of a negative number.

The decimal place will move 5 places to the LEFT.

So:

[tex]0000010\\\\0.00001[/tex]

Now we remember the two, and multiply this by two to get 0.00002.

Hope this helped!

Are the ratios 5:4 and 8:2 equivalent? yes or no

Answers

Answer:

no

Step-by-step explanation:

no its no

Answer:

I believe no

Step-by-step explanation:

I believe it is no but I could be wrong...if I am wrong I am very sorry and hope you can forgive me

Write an algebraic expression for the pair of numbers such that one is 33% more than the other one

Answers

Answer:

x = 1.33y

Step-by-step explanation:

33% is equal to 0.33. If a number x is 33% more than a number y, then x is 133% of y. 133% = 1.33

yeah what the person said above me

The large rectangle below represents one whole. What percent is represented by the shaded area? (URGENT)

Answers

Answer:

40%

Step-by-step explanation:

The whole rectangle is split into 5 parts. Two out of 5 parts are shaded.

Set up a fraction:

[tex]\frac{\text{Shaded}}{\text{Whole}}=\frac{2}{5}[/tex]

Convert into a decimal:

[tex]\frac{2}{5}= 0.4[/tex]

Multiply the decimal by 100 to get the percent:

[tex]0.4*100=40[/tex]

So, forty percent of the rectangle is shaded.

Hope this helps.

The percentage is calculated by dividing the required value by the total value and multiplying by 100.

Required percentage value = a

total value = b

Percentage = a/b x 100

The percentage of the shaded rectangular boxes in the large rectangle is 40%.

What is a percentage?

The percentage is calculated by dividing the required value by the total value and multiplying by 100.

Required percentage value = a

total value = b

Percentage = a/b x 100

Example:

50% = 50/100 = 1/2

25% = 25/100 = 1/4

20% = 20/100 = 1/5

10% = 10/100 = 1/10

We have,

Number of rectangular boxes in the large rectangle = 5

Number of shaded rectangular boxes in the large rectangle = 2

The percentage of shaded rectangular boxes.

= 2/5 x 100

= 2 x 20

= 40%

Thus,

The percentage of the shaded rectangular boxes in the large rectangle is 40%.

Learn more about percentages here:

https://brainly.com/question/11403063

#SPJ2

Select all decimals that are equivalent to (9 x 100) + (2 x 10) + (3x ) + (5 x it).
A. 92.35
B. 920.350
C. 902.35
D. 92.350
D E 920.35
F. 920.035

Answers

Answer:

the answer to the question is letter E I believe

im confused , maybe retype this .

Evaluate the following expression: 2 + 1 x 2 *Please help i don't have that much time*

Answers

Answer:

The answer to your question is 4.

Step-by-step explanation:

Evaluating and simplifying the expression comes to a result of four using the order of operations. Hope this helps you!

The answer is 4 .......

PLEASE HELPP ! i can’t figure out slopes at alll

Answers

Answer:possible

Step-by-step explanation:

Answer is -15/24 you have to find the “rise over run which would be m and in the rise or this case the fall would be -15 and the run would be 24

Write the geometric sequence in function notation.
7, 14, 28, 56, 112, ...
3
Ax) = (7). ()x-1
f(x) = (7) · (2)X-1
O AX) = (7) . (3)X-1
Rx) = (2) (+)*-1

Answers

Answer:

a(n) = 7*2^(n - 1)

Step-by-step explanation:

The first term of the given geometric sequence, 7, 14, 28, 56, 112, ... , is 7, and the common ratio is 2.  Each new term is twice the previous term.

Thus the general formula for the geometric sequence becomes

a(n) = 7*2^(n - 1).

As a check, let's see whether this formula correctly predicts the fourth term (56):  Here n = 4, and so a(4) = 7*2^(4 - 1) = 7*2^3 = 7*8 = 56.  Yes.

Answer:

f(x) = (7) ∙ (2)x–1

Step-by-step explanation:

The first term in the sequence is 7, and the common ratio is 2. Using the explicit formula and writing it in function notation results in f(x) = (7) ∙ (2)x–1.

Two birds start from the same nest and head off in opposite directions. The speed of the first bird is 15mph more than the speed of the second.After 6 hours the two birds are 402 miles apart. Find the speed of each bird?

Answers

Answer:

41 mph26 mph

Step-by-step explanation:

The rate at which distance is increasing between the birds is ...

  r = d/t = (402 mi)/(6 h) = 67 mi/h

This is the sum of the speeds of the two birds, so is 15 mph more than double the speed of the slower bird. That bird's speed is then ...

  (67 -15)/2 = 26 . . . miles per hour

The faster bird is 15 mph faster so is 26+15 = 41 mph.

The slower bird's speed is 26 mph; the faster bird's speed is 41 mph.

41 and 26 miles per hour!

Find the area and perimeter of the shaded region.

Answers

Answer:

Area: [tex]50\pi -100[/tex]

Perimeter: 20[tex]\pi[/tex]

Step-by-step explanation:

If we take half of one of these "pedals" we can see that it is simply 1/4 of a circle with radius 5, subtracted by a triangle. Let's calculate this half-pedal.

[tex]1/4(25 \pi) - 1/2(5* 5)[/tex]

That means 4 pedals is equal to:

[tex]8(1/4(25\pi) - 1/2 (25))[/tex]

[tex]50\pi - 100[/tex]

So.. The area of the shaded region is [tex]50\pi -100[/tex]

Perimeter is even simpler. the half-pedal is just 1/4 of the circumference of the circle. The circumference is just [tex]10\pi[/tex], which means our half pedal is:

[tex]1/4(10\pi )[/tex]

Multiplying by 8, our perimeter is just 20[tex]\pi[/tex].

Perimeter- 20
Area- 50-100

If a person earned $32,000 a year and received $800 raise, what was the percent increase in her salary?

Answers

Answer:

Hey there!

This would be a 2.5% increase in salary.

Using the percent increase formula we find the answer to be a 2.5% increase.

Let me know if this helps :)

Answer:

2.5%

Step-by-step explanation:

((new - original) / original) * 100

328-320 = 8/320 = 0.025

0.025 * 100 = 2.5

w is greater than -1 and less than or equal to 4
Use a only once in your inequality.

Answers

Answer:

-1 + w ≤ 4

Step-by-step explanation:

w is greater than -1 = -1 + w

Less than or equal to 4 = (anything less than or equal to 4)≤4

Answer:

[tex]-1<w\leq 4[/tex]

This shows that w is greater than -1 and less than or equal to 4

, through an inequality.

This is the greater than symbol: <

This is the less than or equal to symbol: [tex]\leq[/tex]

Hope that helped!!!

Please solve these! Read the instructions too!

Answers

just graph the coordinates
this picture is the different quadrants on the graph, whichever quadrant the coordinate is in, is what you put in the 2nd column
for example: (-2,-3) is in the 3rd quadrant
if its not in a quadrant its either on x or y axis, so its just on the line, and i assume you know which axis is which

for number 4. You just do subtraction, and then if its negative, it becomes positive since its asking for the absolute value
for example: |-24-11| > |-35| > 35
نسمينينطمطمطنهطمطمطنطتينطنطن

The t distribution approaches the _______________ distribution as the sample size ___________.

Answers

Answer:

Normal; increases.

Step-by-step explanation:

In Statistics and probability, a student t-distribution also known as the t-distribution refers to the continuous probability distribution which can be used to estimate population parameters when the population variance or standard deviation is not known (unknown) and the sample population is relatively small. The student t-distribution is a statistical distribution which was published in 1908 by William Sealy Gosset.

The t-distribution approaches the normal distribution as the sample size increases because the parameters becomes negligibly smaller.

This ultimately implies that, when the degree of freedom of a t-distribution curve is increased in comparison with that of a normal distribution, the value of probability becomes significantly similar.

Normal; increases!!!
Other Questions
What does the transparency international (TI) lead at global level The period of female fertility during each Month is short For fextilization to take place intercouce must occur eacha) A few hours before ovalation up to 4-5 days after avulationb) Around 24 hours before ovulation up to around. 24 hours after ovulationC) Sometimes between 2-3 days before ovulation upto 2-3 days after ovulationd) Around 4-5 days before ovulation upto a few hours after ovulationE) Around 4-5 days before ovulation upto 4-5. after ovulation. PLEASE HELP!!! WILL BE GIVING BRAINLIEST TO A PERSON!!! What major change in policy did Westmoreland bring about? A. He made the US responsible for most of the military action instead of South Vietnam B. He began a plan of retreat and slow withdrawal from the region C. He switched from the US supplying troops to a South Vietnamese based army D. He recruited nearby nations of Laos and Cambodia to fight for the United States Which answer shows y + 2x < 4x - 3, rewritten to isolate y, and its graph? Which of the following is true about congruent figures?They're the same shape and the same size.They're the same size, but not the same shape.They're not the same shape or size.They're the same shape, but not the same size. Health experts strongly advise patients with hypertension to avoid _____ food amounts of fat. A ball is thrown vertically upward from the ground. Its distance in feet from the ground in t seconds is s(t)=224t16t. After how many seconds will the ball be 720 feet from the ground? The role of the pubic information officer A. includes reporting to the Operations Section Chief during an incident response. B. has become ever more complex as the challenge of delivering messages has evolved. C. is strictly dictated by the National Qualification System position task book. D. is to wait for direction from the incident commander. Question 55 :You use a computer named Wkst1 on a TCP/IP network, which is installed with an application that uses UDP to send a file from your computer to a computer named Wkst2. Data collisions occur, and several data packets are lost during transmission. What will most likely occur as a result of losing the packets during transmission? What is true about the Republican Party during this time ? what does it mean by that "length of an object is m" My sister and I ______________a delicious lunch yesterday (affirmative) The game begins with the player having 20 POINTSThe player rolls 2 six sided dice and the sum of faces on the two dice are calculated. Thats called the players POINT.If the POINT is a 7 or 11, the player wins his POINTIf the POINT is a 2, 3, or 12, the player loses his POINTIf the POINT is anything else, then the game continues by the player rolling the two dice again and again recording and checking the sum untilThe player makes his point (The sum of his dice roll equals his POINT value) and then he wins his POINTThe player rolls a 7 and then he loses his POINT.The player is considered a winner if he can acquire 60 POINTS and is considered a loser when he runs out POINTSAfter the program has finished, display the total number of rolls of the dice that were made in the entire game.In need for python file grade 11 work Significant amounts of phosphorus are found in all locations on Earth except the.. Can someone please help me with both of these questions ? A regulated Natural Monopoly is more likely to advertise freely under which of the following types of regulation?a) price regulationb) profit regulationc) output regulationd) social regulation Based on this stanza, how do the speaker's parents feel about her decision to pursue book publishing? They wish the speaker did not have such an active imagination. They want the speaker to build a large and successful company. They want the speaker to help her family through her work. They wish the speaker would choose a more reliable profession. Coherent light from a sodium-vapor lamp is passed through a filter that blocks everything except for light of a single wavelength. It then falls on two slits separated by 0.490 mm . In the resulting interference pattern on a screen 2.12 m away, adjacent bright fringes are separated by 2.86 mm . For related problem-solving tips and strategies, you may want to view a Video Tutor Solution of Determining wavelength. Part A What is the wavelength of the light that falls on the slits 15. Theythe institute five years ago.(A) have entered(B) entered(C) had entered16. Does the professor _ a lot of experience?(A) has (B) have(C) had The distance from the plane to the building __ meters